dq_computer

87
Chapter 1 Accounting Information Systems and the Accountant 1-1. Take a survey of the students in your class to find out what their parents hold. How many are employed in manufacturing? How many are employed in service industries? How many can be classified as knowledge workers? Employed in Manufacturing Employed in Service Industries Knowledge Workers 1-2. Hiring an employee and taking a sales order are business activities but are not accounting transactions requiring journal entries. Make a list of some other business activities that would not be captured as journal entries in a traditional AIS. Do you think managers would be interested in knowing about these activities? Why or why not? Receiving negative customer feed backs, retirement of the company president, and adding a new supplier of raw materials to the list of valid suppliers are examples of business activities that would not be captured as journal entries in a traditional AIS. Each of these business activities has their own worth to either the management or investors. Information

Upload: jonaslosantas

Post on 16-Jan-2016

53 views

Category:

Documents


6 download

DESCRIPTION

Discussion questions about accounting information systems.

TRANSCRIPT

Page 1: DQ_COMPUTER

Chapter 1 Accounting Information Systems and the Accountant

1-1. Take a survey of the students in your class to find out what their parents hold. How

many are employed in manufacturing? How many are employed in service industries?

How many can be classified as knowledge workers?

Employed in Manufacturing

Employed in Service Industries

Knowledge Workers

1-2. Hiring an employee and taking a sales order are business activities but are not

accounting transactions requiring journal entries. Make a list of some other business

activities that would not be captured as journal entries in a traditional AIS. Do you

think managers would be interested in knowing about these activities? Why or why

not?

Receiving negative customer feed backs, retirement of the company president, and

adding a new supplier of raw materials to the list of valid suppliers are examples of business

activities that would not be captured as journal entries in a traditional AIS. Each of these

business activities has their own worth to either the management or investors. Information

about customer feed backs and new suppliers is important to the production manager. Also,

there are investors who consider the management organization of a company when deciding

whether to invest or not. Therefore, the importance of the information regarding the

retirement of the company president should not be overlooked.

1-3. Advances in IT are likely to have a continuing impact on financial accounting. What

are some changes you think will occur in the way financial information is gathered,

processed, and communicated as a result of increasingly sophisticated IT?

Developments in IT has revolutionized the way financial information is gathered, processed, and

communicated. With the current trend in the development in IT, financial accounting may be

Page 2: DQ_COMPUTER

automated in all phases, with very minimal human intervention. Advances in IT that allow

transactions to be captured immediately, accountants and even the AIS itself will be able to

produce financial statements almost in real-time. Interactive data will allow for information to be

reused and carried seamlessly among a variety of applications or reports.

1-4. XBRL is emerging as the language that will be used to create interactive data that

financial managers can use in communication. How do you think the use of interactive

data might enhance the value of a company’s financial statements?

It will allow the data to be used between programs, allowing for faster access and calculations,

meaning that the company can quickly prepare financial statements at any time.

1-5. Discuss suspicious activity reporting. For example, do you think that such reporting

should be a legal requirement, or should it be just an ethical matter? Do you think that

the majority of SAR activity is illegal, or are these mostly false alarms?

SAR laws now require accountants to report questionably financial transactions to the U.S.

Department of Treasury. It should be a legal requirement because of the ease of committing fraud

through an AIS and the fact that it can be used as a deterrent. Most of the activity is probably false

alarms, but it is better to be safe than sorry.

1-6. Managerial accounting is impacted by IT in many ways, including enhancing CPM.

How do you think a university might be able to use a scorecard or dashboard approach

to operate more effectively?

If ever that the university will use a balanced scorecard to assess its performance, then

the key performance indicators should include the university’s areas of priorities. The

scorecard and dashboard approach allow a university to track and assess the functionality of

its activities and match it against its strategic values. It can trace unfavorable performance to

be able to correct it. This ensures the university has successful internal controls as well. The

dashboard will allow it to be easier to understand given there are many

elements/departments within a university.

Page 3: DQ_COMPUTER

1-7. Look again at the list of assurance services shown in Figure 1-9. Can you think of other

assurance services that CPAs could offer which would take advantage of their AIS

expertise?

CPAs with expertise in AIS may consider rendering services with rather smaller scope for SME.

Business entities which are not giants in their industries may not see it as cost-beneficial the usual

“all-encompassing” service.

1-8. Interview a sample of auditors from professional service firms in your area. Ask them

whether they plan to offer any of the assurance services suggested by the AICPA. Also,

find out if they offer services other than financial auditing and taxation. Discuss your

findings in class.

1-9. This chapter described several career opportunities available to students who combine

a study of accounting with course work in AIS, information systems, and/or computer

science. Can you think of other jobs where these skill sets would be desirable?

One opportunity that I can think of is that accountants could be designing the accounting

system by themselves so as to fully incorporate the specifications that they want the system to

possess.

1-10. This chapter stressed the importance of information technology for understanding

how AISs operate. But is this the only skill valued by employers? How important do

you think analytical thinking skills or writing skills are? Discuss.

For me, no single skill would make an employee precious to the eyes of his employers.

More likely than not, it would always be a combination of skills and professional judgment

that will make your employers see you as an asset. The ability to grasp and analyze things

professionally is a skill needed in dealing with business transactions. Being able to

communicate the analysis either orally or in writing is just as important as possessing

analytical thinking skills.

Page 4: DQ_COMPUTER

Chapter 2 Information Technology and AISs

2-1. Why is it important to view an AIS as a combination of hardware, software, people, data, and

procedures?

Page 5: DQ_COMPUTER

It is best to view an AIS as a combination of hardware, software, people, data, and procedures

because it wouldn’t be a system had it not been for the interaction of these five components.Computer

hardware is probably the most tangible element in this set, but it does not necessarily mean that it is the

most important piece. Computer hardware must work together with the other system components to

accomplish data processing tasks. Without computer software, the hardware would stand idle. Without

data to process, both the hardware and the software would be useless. Without procedures, accounting

data could not be gathered accurately or distributed properly. And finally, without people, the

components mentioned above could not be operated.

2-2. Why is information technology important to accountants?

There are six reasons why IT is important to accountants: (1) Information technology must be

compatible with, and support, the other components of an AIS; (2) Accounting professionals often help

clients make hardware and software purchases; (3) Auditors must evaluate computerized systems; (4)

They are often asked to evaluate the effectiveness and the efficiency of an existing system; (5) IT

profoundly affects the way they work today and the way they will work in the future; and (6)

Understanding how IT affects accounting systems is vital to passing most accounting certification

examinations.

2-3.Why do most AISs try to avoid data transcription?

Data transcription is mostly an inefficient, labor-intensive, time-consuming, costly, and

nonproductive process that has the potential to bottleneck data at the transcription site, embed errors

in the transcribed data, and provide opportunities for fraud, embezzlement, or sabotage.

2-4. Name several types of computer input devices, and explain in general terms how each

onefunctions.

Bar code scanners that are commonly used in most retail stores gather essential inventory data for

the retailer, help avoid human error and they also help speed up the checkout process for customers. In

addition, some retailers are using biometric technology that offers customers convenience and faster

checkouts, and offers retailers savings in transaction costs.

Page 6: DQ_COMPUTER

2-5.How do you feel about red-light cameras? Should cities be allowed to use them? Why or whynot?

Based on the details given by the book about red-light cameras, I see it as an efficient tool in

enforcing discipline among motorists. I disagree with the claims that these red-light cameras pose an

important threat to an individual’s right to privacy, because roads are public property and are therefore

public places, so it is not very logical to demand for utmost individual privacy. Cities should be allowed

to use them as long as the authorities will take the responsibility of placing these cameras on

appropriate places.

2-6.Identify the three sections of a CPU, and describe the functions of each component. How are

microprocessor speeds measured? Why are such speeds rarely important to AISs?

The three components of a CPU are: primary memory, ALU, and control unit. The purpose of

primary memory is to store data and program instructions temporarily for immediate processing and

execution. The arithmetic-logic unit (ALU) portion of microprocessor chips performs arithmetic tasks

(such as addition and multiplication), as well as logic tasks (such as comparisons). In contrast, the control

unit of the processor supervises the actual data processing.

2-7. Identify several types of printers. What are the advantages and disadvantages of each type?

2-8.What is the function of secondary storage? Describe three types of secondary storage media,and

describe the advantages and disadvantages of each type.

Secondary storage devices are permanent media that maintain accuracy and integrity of data, yet

permit the system to access and modify information quickly and easily.

Type of Printer Advantages Disadvantages

Dot-matrix Inexpensive and can print

multipart (‘‘carbon’’) paper

Lower resolution as compared to ink-

jet printers; cannot print in colors

Ink-jet Higher resolution as compared to

dot-matrix; can print in colors

Printing speeds are lower than that of

laser printers

Laser Superior speeds and printing

quality

Highest cost among the three

Page 7: DQ_COMPUTER

2-9.What is image processing? How is image processing used in AISs?

Image processing allows users to store graphic images in digital formats on secondary storage

media. Commercial users of image processing include (1) insurance companies that use image

processing to store claims forms and accident reports, (2) banks that use image processing to store

check images, (3) hospitals that use image processing to store medical diagnostic scans, and (4) the

internal revenue service that uses image processing to capture and store tax return data.

2-10.What are data communication protocols? Why are they important?

Type of secondary

storage media

Advantages Disadvantages

CD-ROMs Ideal for storing large amounts of

accounting data or reference materials;

data transfer rates are also very fast; wear

and tear is minimal; and the worm

characteristic of CD-ROMs and CD-Rs make

them useful for archiving files securely

Worm media cannot be updated

because new information cannot

be written on them once they

have been encoded.

DVDs Can archive large amounts of data; single,

light-weight, reliable, easily transportable

medium; and newer DVDs are writeable

and even rewriteable.

None mentioned

Flash Memory Can store gigabytes of data in an erasable

format; because data transfer rates are

high and the devices themselves are

compact, they are particularly useful to

accountants for creating backups of

important files and transporting them

offsite; and costs are low.

None mentioned

Page 8: DQ_COMPUTER

A data communications protocol refers to the settings that provide the format for sending and

receiving stations in all data communications applications. The importance of protocols lies on the uses

of the two types of protocols. The two common protocols are TCP (transmission control protocol), which

networks commonly used for e-mails, and HTTP (hypertext transmission protocol), which networks

commonly used for Web pages.

2-11.What are local area networks? What advantages do LANs offer accounting applications?

LANs consist of microcomputers, printers, terminals, and similar devices that are connected

together for communications purposes. The advantages that LANs offer accounting applications are:

facilitating communications; sharing computer equipment; sharing computer files; saving software costs;

and enabling unlike computer equipment to communicate with one another.

2-12.What is client/server computing? How does it differ from host/mainframe computing? What are

some of the advantages and disadvantages of client/server systems?

Client/server computing is a set of three interacting components (a presentation component, an

application–logic component, and a data-management component) that serves as an alternative to

mainframe networks. Mainframe systems normally centralize everything, including the control of the

system, while client/server applications distribute data and software among the server and client

computers of the system. Some of the advantages of client/server computing include the flexibility of

distributing hardware, software, data; processing capabilities through-out a computer network; and

reduced communication costs. One disadvantage of a large client/server system is that it must maintain

multiple copies of the same databases, which it then stores on its various regional servers.

2-13.What are the names of some current cloud computing vendors other than those discussed inthe

text? Do you think that all firms should use cloud vendors, or are there some reasons why they should

be avoided?

Some of the current cloud computing vendors are Green Pages, Amazon Web Services (AWS),

VMware, Microsoft, Bluelock, Citrix, Joyent, Terremark, Salesforce.com, Savvis, and Rackspace. Firms

should weigh the costs and the related benefits of resorting to cloud computing and it is only when the

benefits exceed the costs that they should choose cloud computing.

Page 9: DQ_COMPUTER

2-14.What are windowing operating systems, multitasking operating systems, and graphical user

interfaces? Why are they useful to AISs?

An operating system (OS) is a set of software programs that helps a computer, file server, or

network run itself and also the application programs designed for it. The term graphical user interface

(GUI) refers to one or more visual computer screens that enable an end user to communicate with a

computer—typically by selecting items from menus or clicking on choices using a computer mouse.

Computer programs that did not use GUIs typically were command-driven systems that required users

to memorize and type in system commands and instructions.

2-15. Name some general classes of application software. What tasks do each of the software classes

perform?

Classes of Application Software Tasks performed

Personal productivity software Allows users to create documents and reports, worksheets, files

and databases, and manage personal finance

Accounting software Performs familiar tasks such as preparing payrolls, maintaining

accounts receivable files, executing accounts payable tasks,

controlling inventory, and producing financial statements

Communications software Allows separate computers to transmit data to one another

ERP software Enables businesses and government agencies to transmit and

manipulate financial data on an organization-wide basis

2-16.What are computer programming languages? Name some specific languages and describe briefly

an advantage of each.

A programming language is a language by which application software developers write instructions

that computers can understand and execute. Examples are: C++(favored for its ability to manipulate

data at the bit level), Visual Basic (favored for creating Windows-like user interfaces), HTML (an editing

language favored for creating Web pages), and Java (favored for its ability to run on many different

types of computers).

Page 10: DQ_COMPUTER

Chapter 3 Data Modeling

3-1. Why are databases important to accounting information systems? Describe some concerns and

explain why each one is important.

Nearly every accounting system that influences financial reports involves the extensive use of

databases. Firms rely on these databases to make key business decisions and conduct their day-to-day

operations. Concerns relating to the significance of databases to AIS are the following: critical

information (the information stored in an organization’s databases is sometimes its most important and

valuable asset); volume (many firms’ databases are truly enormous); distribution (the databases of

some organizations are centralized); privacy (databases often contain sensitive information);

irreplaceable data (the information contained in most accounting databases is unique to the

organization that created it and is typically priceless); need for accuracy (The data stored in databases

must be complete, comprehensive, and accurate); and internet users (databases store information

related to product information for online catalog sales, e-mails, product registration data, employment

opportunities, stock prices, and so on).

3-2.What is the hierarchy of data in databases? Provide an example for a particular accounting

information.

Page 11: DQ_COMPUTER

The hierarchy of data in a database is presented as: data field→record→file→database.

The first level in the data hierarchy is a data field, which is information that describes a person, event, or

thing in the database. In a payroll file, for example, data fields would include employee names,

employee identification numbers, and pay rates for the employees. At the second level, data fields

combine to form a complete record. A database record (also called a tuple) stores all of the information

about one entity (i.e., person, event, or thing). For example, all of the information about one inventory

part in an inventory file, one employee in a payroll file, or one customer in a customer file. At the third

level of the data hierarchy, a set of common records forms a file, or using database and Microsoft Access

terminology, a table. Thus, a file or table contains a set of related records—for example, a set of

customer records or inventory records. Master files typically store permanent information—for

example, part numbers, part descriptions, and location codes for the individual records in an inventory

parts master file. Transaction files typically store transient information—for example, inventory

disbursements and replenishments for a specific period. Finally, at the highest level, several tables

create a complete database (i.e., a collection of tables that contain all of the information needed for an

accounting application). In an inventory application, for example, the database might contain a part-

number master table, a supplier table, a price table, an order transaction table, and so forth, as well as

several other tables that would help end users organize, access, or process inventory information

efficiently.

3-3.What are primary keys in accounting databases and what purpose do they serve?

The primary key is the data field in each record that uniquely distinguishes one record from another

in a database table. Primary keys are required for every record in a database and they are unique. End

users and computer programs use primary keys to find a specific record

3-4.Name some specific accounting files and a potential primary key for each one.

Example of an Accounting File Potential Primary Key

Accounts Receivable Customer Account Number

Accounts Payable Creditor Account Number

Vendor History Vendor Code

Page 12: DQ_COMPUTER

3-5. Describe each of the following database issues that are relevant to accounting systems, and give

an example of each: (1) data integrity, (2) transaction accuracy and completeness, (3) concurrency

processing, and (4) security.

Database Issue Description Example

data integrity These data integrity controls are

designed by database developers

and are customized for different

applications.

tests for data completeness,

conformance to the data type

specified for the data field, valid

code tests, and reasonableness

tests

transaction accuracy and

completeness

AISs need transaction controls to

ensure that the database system

performs each transaction

accurately and completely

Database systems maintain an

auditable log of transactions to help

users deal with partially-executed

transactions.

concurrency processing Without concurrency controls, it is

possible for two or more users to

access the same record from the

same table at the same time which

creates problems.

Database systems include locking

mechanisms that do not allow

multiple users to access the same

record at the same time to prevent

conflict in the information obtained

by users concurrently accessing the

same record at the same time.

security A key security feature of any

database is a set of backup

procedures that enable the

organization to recreate its data if

the original copies are lost or

damaged.

Backup security, protecting the

system from unauthorized access,

use of view controls

3-6.What is the REA model? How does REA differ from more traditional accounting views of data

collection and storage? Hint: would a traditional accounting database store data about personnel

matters?

Page 13: DQ_COMPUTER

REA is an acronym for resources (R), events (E), and agents (A). The basic assumption of this model

is that business events affect firm resources and involve agents who participate in the event. When

creating a database using an REA approach, a system designer will try to record all events that are

relevant for management decision making in the database, whether they are business or economic

events. On the other hand, traditional accounting database keeps record of economic events only,

disregarding those that are not financial in nature.

3-7.What are database cardinalities? Give some examples of cardinalities for an accounting

application other than sales.

Cardinalities describe how the entities in a database are related. For example, a company where

each company laptop computer is assigned to only one manager, and every manager is assigned only

one computer. This is actually a one-to-one relationship. If this relationship will be depicted in a

database, each record in one entity will always be associated with one (and only one) record in the

associated entity.

The next example is a many-to-many association. For example, a company has a policy of

purchasing the same types of inventory from multiple suppliers. Management may choose to do this to

ensure that they get the best prices or avoid becoming dependent on a single supplier. Under such a

policy, each Vendor record is associated with one or many Inventory records and each Inventory record

is associated with one or many Vendors.

3-8.What is an entity-relationship diagram? What can you determine about an organization from

examining an E-R diagram?

An entity-relationship diagram is a graphical depiction of the entities and their relationships. When

using a REA modeling approach, the diagrams are arranged as events that occur in temporal sequence.

Therefore, a reader can quickly see the main business events and the order in which events occur.

3-9. Suppose that a data modeler creates a database that includes a Sales table and a Salesperson

table. Would you be likely to need a relationship table to link these two entities? Why or why not?

These tables need to be linked because a data field in Salesperson, for example, Saleperson ID, is

needed in the Sales table. The Salesperson ID data field, which is the primary key in the Salesperson

table will be the foreign key in the Sales table.

Page 14: DQ_COMPUTER

3-10.What is the process of normalization? What levels are there, and why do database developers

seek to normalize data?

Normalization is a methodology for ensuring that attributes are stored in the most appropriate

tables and that the design of the database promotes accurate and non-redundant storage of data. The

process of normalization can result in the creation of new tables to include in the database.

The following are the three levels of normalization as discussed in the book:

First Normal Form. A database is in first normal form (1NF) if all of a single record’s attributes (data

fields) are singular. That is, each attribute has only one value. This corrects the problems relating to

nonsingular attributes.

Second Normal Form. A database is in second normal form if it is in first normal form and all the

attributes in each record depend entirely on the record’s primary key. This corrects the problems

relating to data redundancy, insertion anomaly, and deletion anomaly.

Third Normal Form. A database is in third normal form if it is in second normal form and contains no

transitive dependencies.

Page 15: DQ_COMPUTER

Chapter 4 Organizing and Manipulating the Data in Databases

4-1.This chapter described how to create tables and records in Microsoft Access. What other database

management are available? Use the internet to learn more about these systems.

Other examples of database management system are Alpha 5, Query, Filemaker Pro, and Lotus

Approach.

4-2. Identify the different data types available for creating data fields in Microsoft Access. Similarly,

identify the different types of numbers (e.g., long integer) you can use if you define a field as a data

type.

The different data type in Access 2013 are: Short text, Long text, Number, Date/Time, Currency,

AutoNumber, Yes/No, OLE Object, Attachment, Hyperlink and Calculated. The different types of

numbers include byte, integer, long integer, single, double, replication ID, and decimal.

4-3. Create a Salesperson table and a Customer Order table using the data in Figure 3-11. Create

records for each table using the data provided. Add one more Salesperson record with your own

name and an employee number of your own choosing. Also add at least one customer order with your

number as the salesperson. Finally, create a relationship for the two tables. Create a hard-copy

documentation of your work.

Page 16: DQ_COMPUTER

4-4. What are database management systems? Are they the same as databases? Why are DBMSs

classified as software and not hardware?

A database management system (DBMS) is a software system that enables users to create database

records, delete records, query (i.e., select subsets of records for viewing or analysis), alter database

information, and reorganize records as needed. This section of the chapter explains

how to perform some of these tasks in greater detail. A DBMS is not a database. Rather, a DBMS is a

separate computer program that enables users to create, modify, and utilize a database of information

efficiently, thus allowing businesses to separate their database system operations from their accounting

system applications. This enables organizations to change record structures, queries, and report formats

without also having to reprogram the accounting software that accesses these database items. It also

enables businesses to upgrade either system independently of other systems.

4-5. What are data definition languages (DDLs)? How are they related to DBMS?

The data definition language (DDL) of a DBMS enables its users to define the record structure of any

particular database table tab (i.e., the individual fields that each record will contain). DDL is the language

that DBMSs use to create the data dictionaries.

4-6. Why do database developers link tables together? How is this done using Access?

Database developers link tables together in order to establish relationships between or among

tables. This is done in Access by following two steps as follows:

Step 1—Select tables. Select ‘‘Relationships’’ from the ‘‘Database Tools’’ tab. From the tables listed on

the left of your screen, right click on the table you wish to link (tblPayrollMaster File: Table), drag it into

the Relationships window, and release the mouse. Do the same with the tblDepartments: Table. Boxes

for the two tables in the Relationships window will appear, but there will not be a line drawn between

the two tables.

Step 2—Link the tables. To link the two tables together, drag and drop the Department Code name from

either table to the similar name in the other table. When you do, you should also see the Edit

Relationships dialog window. This window enables you to enforce referential integrity. Check this box. In

the context of this example, referential integrity is a control that prohibits users from creating employee

records with references to nonexistent departments. It does not affect your ability to create a

department with no employees, however.

Page 17: DQ_COMPUTER

4-7.What is data validation? Why is it important? Give some examples of how to validate data inputs

using Access.

Data validation involves the creation of rules such as record-structure-property of a data field that

serves as data-entry controls. Validation rules are important because they limit the user from entering

data that do not satisfy the desired property of data in a certain data field. An example of using

validation rule is “Between 1 and 100”, which is a validation rule used to prevent the user from entering

data outside the given range.

4-8. What are data manipulation languages? How are these languages related to database

management systems? How are these languages related to databases?

Data manipulation language (DML) is the proprietary programming language that a particular DBMS

uses to retrieve, process, and store data. DML is related to DBMSs because an entire user programs may

be writ- ten in the DML or, alternatively, selected DML commands can be inserted into programs that

are written in universal languages. DML is related to databases because it is through DML that a

particular database perform functions relevant to users, such as retrieval of data through queries.

4-9.What is SQL? How is SQL like an Access query? How is it different?

SQL (Structured Query Language) is the standard query language for both mainframe and

microcomputer DBMSs.

4-10.What is online analytical processing? How is OLAP related to databases? What is a pivot table

and how are pivot tables and OLAP related?

OLAP involves using data-extraction tools to obtain complex information that describes what

happened and also why it happened. Pivot tables are two dimensional statistical summaries of database

information.

4-11.What is the difference between sorting records and indexing records in a database?

Sorting records are unlike indexing records in that sorting physically rewrites records on disks

whereas indexing does not.

4-12.What is data mining? How is data mining useful to profit-seeking companies? What are some

accounting uses of data mining?

Page 18: DQ_COMPUTER

Data mining is a set of data analyses and statistical tools to detect relationships, patterns, or trends

among stored data. Data mining is useful to profit-seeking companies because data mining tools enable

companies to perform tasks that make their business functions, such as Sales and Internal Control more

efficient. One accounting application of data mining is for auditors to use data mining to detect credit

card anomalies or suspicious behavior. For example, fraudulent credit card transactions may follow a

pattern, such as an increase in the total amount of purchases immediately following a credit card theft

or products with special characteristics (such as ones that can easily be sold). Another application is for

investors to use data mining tools to predict corporate bankruptcies. The use of data mining is also

expanding in the audit profession.

4-13.What is cloud computing? How will it influence accounting systems?

Cloud computing is an Internet-based computing that allows users to use business applications on

the Internet. Cloud computing influence accounting systems both positively and negatively. The chief

advantage of cloud computing is that it is less expensive as compared to purchasing software. On the

other hand, cloud computing has security as its major drawback.

4-14.What are data warehouses? How are they like databases? How do they differ from databases?

Where feasible, it often makes sense to pool the data from separate applications into a large,

common body of information called a data warehouse. Data warehouse are like databases because they

store information about the same subject. They are different because data warehouses typically store

older information which are clean and free from errors, while databases are usually updated and are not

necessarily error-free.

4-15.Why would a company be interested in creating a data warehouse? Why would a company not

be interested in creating a data warehouse?

Companies may be interested in creating data warehouses if they intend to make organizational

information corporate-wide. Data warehouses can also be used on predicting future things such as sales

for a new product. But building a data warehouse is a complex task, and the benefits to be derived by a

company could sometimes not exceed the costs of creating one.

Page 19: DQ_COMPUTER

Chapter 5 Database Forms and Reports

5-1. What are some of the advantages and disadvantages of database forms?

Database forms enable you to add records to a database table, modify the data in existing records

of a table, or simply view the data in a table. The ability to customize a form, provide explanations for

data-entry fields within forms, and create convenient tab orderings are especially useful features for

ensuring valid data entry. In commercial environments, the database developer is rarely the same

person who enters data in database tables on a daily basis.

5-2.Would you rather use a form or a datasheet for entering data into a database table? Why?

In commercial environments, the database developer is rarely the same person who enters data in

database tables on a daily basis. Anything that the developer can do to make this job more convenient

Page 20: DQ_COMPUTER

and straightforward for data-entry personnel helps avoid errors, streamlines the data-entry process, and

saves time.

5-3. To create a form, would you rather use the Form Wizard in Access or create the form

fromscratch? Why?

Form Wizard in Microsoft Access can be used to rapidly design and create custom forms. These

custom forms usually fit the needed design, so I would rather use the Form Wizard rather than starting

from scratch.

5-4. What is a subform? Why do forms have subforms? How do you create subforms in Access?

A subform is a form within a form that displays data related to the information in the main form.

Forms have subforms because a subform displays data related to the information in the main form. To

create a subform in access, first, open the form (from which you want to create a subform) in design

view and use your mouse to extend the size of the details section of the form to make room for the

subform. Then, click on the Subform icon in the Toolbox and use your mouse to draw a rectangle in the

detail section of your form. This procedure causes Access to launch the Subform Wizard, which will ask

you for settings like ‘‘which table do you want to use for the subform,’’ ‘‘how do you want the data to

appear in the subform,’’ and so forth. Click finish after you are done with choosing the settings.

5-5.Why do database developers customize forms? Why isn’t it sufficient to use the form asinitially

created by the Form Wizard?

In design mode, your resulting form and subform will not look exactly like the one that you like. For

example, you will probably have to resize the outer dimensions of the subform to fit the data and

perhaps reword the text in the label at the top of the subform.

5-6.What is the purpose of a database report? What information do such reports contain?

For example, you will probably have to resize the outer dimensions of the subform to fit the data

and perhaps reword the text in the label at the top of the subform. Reports can be simple documents

that display only the contents of a table or complex outputs that combine the information from several

tables and show selected subsets of database information useful for decision-making.

Page 21: DQ_COMPUTER

5-7.The chapter suggested that it is important to design the format of a report before creatingthe

report itself. Do you agree with this suggestion? Why or why not?

Yes, I agree. Designing the format before creating the report makes you more directed when you

are in the actual creation of the report.

5-8. Do you think that we will still use hardcopy reports in the future, or will they be replacedwith

softcopy ones? Defend your answer.

There will always be people who respond to hardcopy reports better than with softcopy ones so I

think there will still be hardcopy reports in the future.

5-9. Would you rather use the Report Wizard to create the format of a report or design oneyourself

from scratch? Why?

A Report Wizard is a tool available in Access that allows for rapid creation of reports. I would rather use

the Report Wizard to create the format of a report, unless the design that I need is impossible to be

done using a Report Wizard.

5-10. What is a calculated field in a report? Provide some examples. Why do reports containcalculated

fields?

Calculated fields are fields in a database that are calculated using other fields in the database. A

Sales field calculated by multiplying data in a Units Sold field and Unit Selling Price field. Reports contain

calculated fields because some of the data fields that is needed to be shown in a report are not recorded

as normal fields in a database.

5-11. Why don’t databases store calculated fields as normal fields in database tables? Do you

thinkthey should?

The reason why calculated fields are not stored in database tables as normal fields is that they are

usually needed only for the purpose of generating reports and it will be more practical to not store them

as normal fields. Sometimes, the user wants the calculated field to appear in the detail section of a

report, while at other times, the user wants group or grand totals to appear in the group footer or the

report footer sections of your report.

5-12. Why are calculated fields created with database queries? Why not create them directly

withreports?

Page 22: DQ_COMPUTER

It is usually easiest to create calculated fields using queries rather than tables for the underlying

data.

Chapter 6

6-1. Why is documentation important to accounting information systems? Why should accountants be interested in AIS documentation?

Accountants do not need to have the ability to program complex systems, but it is important for them to understand the documentation that describes how processing takes place. Documentation includes the flowcharts, narratives, and other written communications that describe the inputs, processing, and outputs of an AIS. Documentation also describes the logical flow of data within a computer system and the procedures that employees must follow to accomplish application tasks. It also helps accountants to:

Depicting how the system works Training users Designing new systems Controlling system development and maintenance costs Standardizing communications with others Auditing AISs Documenting business processes Complying to the law Establishing accountability

Page 23: DQ_COMPUTER

6-2. Distinguish between document flowcharts, system flowcharts, data flow diagrams, and program flowcharts. How are they similar? How are they different?

A document flowchart traces the physical flow of documents through an organization—that is, the flow of documents from the departments, groups, or individuals who first created them to their final destinations.

System flowcharts concentrate on the computerized data flows of AISs. Thus, a system flowchart typically depicts the electronic flow of data and processing steps in an AIS

Data flow diagrams are tools for analyzing an existing system or as a planning aid for creating a new system.

Program flowcharts describe the processing logic of each application program.

Differences:

Processes on DFDs can operate in parallel (at-the-same-time)Processes on flowcharts execute one at a time

DFDs show the flow of data through a systemFlowcharts show the flow of control (sequence and transfer of control)

Processes on a DFD can have dramatically different timing (daily, weekly, on demand)Processes on flowcharts are part of a single program with consistent timing

6-3.What are document flowcharts? How does a document flowchart assist each of the following individuals: (1) a systems analyst, (2) a systems designer, (3) a computer programmer, (4) an auditor, and (5) a data security expert?

A document flowchart traces the physical flow of documents through an organization—that is, the flow of documents from the departments, groups, or individuals who first created them to their final destinations.A document flowchart is used in designing or evaluating an accounting information system.

1. A systems analyst uses it when evaluating a system to see if each department is receiving the necessary data and that unnecessary data are not transferred.

2. A system designer uses it when there is interest in improving or replacing an existing system.

3. A computer programmer can use a document flowchart when preparing system flowcharts. 

4. An auditor uses it to help define, follow, and evaluate an audit trail. 5. An internal data security expert uses it to indicate weaknesses in internal control and data

control.

Page 24: DQ_COMPUTER

6-4. Flowcharting is both an art and a science. Guidelines can be used to make better flowcharts. What are these guidelines for document, system, and data flow diagram flowcharts?

Guidelines for data flow diagram:

1. Creating DFDs is as much art as science. The following rules can help you make them easier to interpret and assist you in avoiding simple errors.

2. Avoid detail in high-level DFDs (i.e., in levels 0 and 1). Where appropriate, combine activities that are performed at the same place or same time or that are logically related.

3. As a general rule, each logical DFD should contain between five and seven processing circles (or bubbles). This guideline helps you simplify the diagrams and avoid showing too much detail in high-level DFDs.

4. Different data flows should have different names to avoid confusion related to the data produced and used by different processes.

5. Unless they are outside the system or used for archiving, all data stores should have data flows both into and out of them. Thus, an internal file symbol that lacks both of these data flow lines typically involves a diagramming error.

6. Even if a file is temporary, it is usually desirable to include it in a DFD.7. Classify most of the final recipients of system information as external entities. 8. Classify all personnel or departments that process the data of the current system as

internal entities.CHAPTER 6 / Documenting Accounting Information Systems 177 9. Display only normal processing routines in high-level DFDs. Avoid showing error

routines or similar exception tasks.10. Where several system entities perform the same task, show only one to represent

them all. This rule also applies when system personnel perform the same task at different locations of the organization—for example, at different plants.

Guidelines for document flowcharts:

1. Identify all the departments that create or receive the documents involved in the system. Use vertical lines to create ‘‘swim lanes’’ to separate each department from the others.

2. Carefully classify the documents and activities of each department, and draw them under their corresponding department headings.

3. Identify each copy of an accounting document with a number. If multiple-copy documents are color-coded, use a table to identify the number–color associations.

4. Account for the distribution of each copy of a document. In general, it is better to over-document a complicated process than to under-document.

5. Use on-page and off-page connectors to avoid diagrams with lines that cross one another.6. Each pair of connectors (a from and a to connector in each pair) should use the same

letter or number.

Page 25: DQ_COMPUTER

7. Use annotations if necessary to explain activities or symbols that may be unclear. These are little notes to the reader that help clarify your documentation.

8. If the sequence of records in a file is important, include the letter ‘‘A’’ for alphabetical, ‘‘N’’ for numeric, or ‘‘C’’ for chronological in the file symbol. As indicated in guideline 7, you can also include a note in the flowchart.

9. Many flowcharts in practice use acronyms (e.g., GRF or PHF in the preceding examples). To avoid confusion, use full names (possibly with acronyms in parentheses) or create a table of equivalents to ensure accuracy in identifying documents.

10. Consider using automated flowcharting tools. See the section of this chapter on CASE tools for more information.

Guidelines for system flowcharts:

1. System flowcharts should read from top to bottom and from left to right. In drawing or reading such flowcharts, you should begin in the upper-left corner.

2. Because system flowcharting symbols are standardized, you should use these symbols when drawing your flowcharts—do not make up your own.

3. A processing symbol should always be found between an input symbol and an output symbol. This is called the sandwich rule.

4. Use on-page and off-page connectors to avoid crossed lines and cluttered flowcharts. 5. Sketch a flowchart before designing the final draft. Graphical documentation software

tools (discussed shortly) make this job easier. 6. Add descriptions and comments in flowcharts to clarify processing elements. You can

place these inside the processing symbols themselves, include them in annotation symbols attached to process or file symbols, or add them as separate notes in your systems documentation.

6-5. What are the four symbols used in data flow diagrams? What does each mean?

A DFD usually comprises of four components. These four components can be represented by four simple symbols. These symbols can be explained in detail as follows: External entities (source/destination of data) are represented by squares; Processes (input-processing-output) are represented by rectangles with rounded corners; Data Flows (physical or electronic data) are referred to by arrows; and finally, Data Stores (physical or electronic like XML files) are presented by open-ended rectangles.

6-6. Why are data flow diagrams developed in a hierarchy? What are the names of some levels in the hierarchy?

Data flow diagrams developed in a hierarchy helps decide what system resources to acquire, what activities employees must perform to run these systems, and how to protect and control these systems after they are installed. The hierarchical DFD typically consists of a top-level

Page 26: DQ_COMPUTER

diagram (Level 0) underlain by cascading lower level diagrams (Level 1, Level 2…) that represent different parts of the system.

6-7. Look at the process map in Figure 6-5. Trace the steps in the order fulfillment process. Do you think this figure is more helpful than a narrative would be in understanding the flow of events in the process?

First, the time card data of an employee is being sent to the payroll master file. Then, the system will process it in order to update the employee’s data and help process paychecks and payroll reports for management and government. If there are some changes about the employee’s data (employee’s status and rate data) coming from the human resource department, the system will also update it. The figure presented in Figure 6-5 is more helpful in understanding the flow of events in the process for it shows the whole picture of how a system works.

6-8.What is the purpose of a decision table? How might decision tables be useful to accountants?

A decision table is a table of conditions and processing tasks that indicates what action to take for each possibility. Sometimes, decision tables are used as an alternative to program flowcharts. More commonly, they are used in conjunction with these flowcharts. Decision tables are useful to accountants in a sense that they summarize the processing tasks for a large number of conditions in a compact, easily understood format. This increases system understanding, resulting in fewer omissions of important processing possibilities. Decision tables also serve as useful documentation aids when new data processing conditions arise or when changes in organizational policy result in new actions for existing conditions. This advantage is particularly important to AISs because of organizational concern for accuracy and completeness in processing financial data.

6-9.What are CASE tools? How are they used? How do CASE tools create documentation for AISs? If you were a systems analyst, would you use a CASE tool?

CASE (computerassisted software engineering) tools automate such documentation tasks as drawing or modifying flowcharts, drawing graphics and screen designs, developing reports, and even generating code from documentation. Thus, CASE tools are to flowcharts what word processors are to text documents

6-10.What is end-user computing? Why is documentation important to end-user computing? What guidelines should companies develop to control end-user computing?

End-user computing refers to the ability of non-IT employees to create computer applications of their own. End-user applications often perform mission-critical functions for busy organizations. In many cases, the outputs of user-developed spreadsheets and database applications find their way into financial systems and ultimately influence an organization’s financial statements. Thus, end users should document their applications for many of the same reasons that professionals

Page 27: DQ_COMPUTER

must document applications. Managers, auditors, and other system users need to understand how user-developed systems work in order to prevent errors created by these systems. In addition, if an employee leaves an organization, other employees need to be able to use these applications and interpret their outputs.

Guidelines in controlling end-user computing:

Formally evaluate large projects Adopt formal end-user development policies Formalize documentation standards Limit the number of employees authorized to create end-user applications Audit new and existing systems

Chapter 7

7-1. As you might imagine, the chart of accounts for a manufacturing firm would be different from that of a service firm. Not surprisingly, service firms differ so much that software now exists for almost any type of firm that you could name. Think of yourself as an entrepreneur who is going to start up your own business. Now, go to an Office Depot, Staples, or a similar office supply store (or search online) to find at least two different software packages that you might use for the type of firm you are going to start up. What does the Chart of Accounts include? Are both software packages the same? What are the differences between the packages?

7-2. What are the purposes of accounting codes? How are they used? Bring to class some examples of codes used by manufacturing firms, accounting firms, and merchandising firms.

The main purpose of accounting codes is to identify accounts. It is also used to record transactions systematically.

Example of Accounting Codes

Page 28: DQ_COMPUTER

Block Assigned Type of Account

100-199 Assets

200-299 Liabilities

300-399 Owner's Equity

400-499 Revenues

500-599 Costs of Goods Sold

600-699 Expenses

700-799 Other Revenue

800-899 Other Expenses

7-3. What are some typical outputs of an AIS? Why do system analysts concentrate on managerial reports when they start to design an effective AIS? Why not start with the inputs to the system instead?

Among the outputs of an AIS are: (1) reports to management, (2) reports to investors and creditors, (3) files that retain transaction data, and (4) files that retain current data about accounts (e.g., inventory records). Perhaps the most important of these outputs are the reports to management because these reports aid decision-making activities. As you might imagine, the formats of these various reports might be very different. These reports might be hard-copy (paper) reports, soft-copy (screen) reports, e-copy (CD and other electronic media) reports, or audio outputs. If a manager queries a database system, the monitor shows the requested data and the system produces a hard-copy report only upon demand. Graphics enhance reports in any form. Many reports today appear on company Web sites. While Web page design is beyond the scope of this book, it is important to recognize that the rules for preparing good reports apply to Web page reports as well as hard-copy and other multimedia reports.

7-4. What are some criteria that systems designers should consider when developing managerial reports for an AIS? How do system designers know what to include on reports?

AIS reports should be consistent: (1) over time, (2) across departmental or divisional levels, and (3) with general accounting practice. System designers must also know that good managerial reports always contain fundamental identification, including headings (company name, organizational division or department, etc.), page numbers, and dates.

7-5. Visit a local business and collect some examples of source documents used in an AIS. For each source document you collect, discuss its purpose(s). Are different source documents

Page 29: DQ_COMPUTER

required for manufacturing firms versus merchandising organizations? Are all the business’ source documents paper based?

A purchase order (PO) is a commercial document and first official offer issued by a buyer to a seller, indicating types, quantities, and agreed prices for products or services.There are some differences in purchase order of a manufacturing firm on that of a merchandising organization. But both still contains the necessary elements of a purchase order. Most of source documents are paper-based but it doesn’t mean it should always be made of paper. Some source documents are electronic, like electronic record of the hours worked by an employee, as entered into a company's timekeeping system through a smart phone.

7-6. This chapter discussed many inputs to an organization’s sales process. What are the specific data items needed to add a new customer and record a sales order?

Sales order Sales invoice Remittance advice Shipping notice Debit/credit memoranda

7-7. How does a data flow diagram for the sales process differ from a system flowchart describing that process?

The data flow diagram of a sales process shows where data came from and went on. On the other hand, a system flowchart describes a how a sales processing system works.

7-8. How are the inputs and outputs of the purchasing process likely to be different for a restaurant versus an automobile manufacturer?

The inputs and outputs of the purchasing process of a restaurant will be likely different of that an automobile manufacturer in terms of number of inventory (raw food ingredients vs. materials used in doing an automobile) etc.

7-9. Explain the term business without boundaries. How is this changing the nature of organizations and who accomplishes various business processes?

Business without boundaries means companies no longer have all of their employees in one location, working on various business processes such as HR, accounting, production, and others. Employees may be located anywhere in the world, and they are. The result is a new dimension to outsourcing called offshoring—moving jobs offshore—to countries such as India, China, Canada, Mexico, or Malaysia.It forces businesses to adapt to the change and all business processes are under a great deal of scrutiny by managers and management accountants as

Page 30: DQ_COMPUTER

companies become more strategically oriented toward revenue generation and more vigilant about managing costs.

7-10. What do we mean when we say companies are offshoring business processes?

It simply means that important processes of an organization are being done by employees located in another area or countries. For example, a US-based organization offshore its payroll process to another entity located here in the Philippines. The entity based here in Philippines will be the one who will do the payroll process for the US-based organization.

7-11. Some businesses choose offshoring to solve the issue of expertise, especially for IT personnel. These companies claim they simply cannot find enough qualified employees in the United States to do certain technology jobs. Do you agree with this assessment? Why or why not?

I agree. There are benefits or advantages derived in doing such measures. Such benefit derived is a lower cost in offshoring to another country such jobs than hiring employees in US that aren’t qualified enough for the job. Minimizing cost is one of the objectives of a business enterprise.

7-12. Search the Web for unusual and interesting uses of RFID tags. Find at least two that are unusual and share those with your classmates.

RFID tags come in a wide variety of shapes and sizes; they may be encased in a variety of materials:

Animal tracking tags, inserted beneath the skin, can be rice-sized. Tags can be screw-shaped to identify trees or wooden items. Credit-card shaped for use in access applications. The anti-theft hard plastic tags attached to merchandise in stores are also RFID tags. Heavy-duty 120 by 100 by 50 millimeter rectangular transponders are used to track

shipping containers, or heavy machinery, trucks, and railroad cars.

7-13. Discuss the privacy issues created by the use of RFID tags. Do you support the use of RFID tags for personal ID, customer relationship management, and inventory tracking? Why or why not?

No, unless there will be provisions that will limit the use of RFID tags when it comes to personal data of a person. It should be noted that even RFID tags promote the business; it should always not be at the expense of persons working for the business.

Page 31: DQ_COMPUTER

Chapter 8

8-1. The resource management process includes events associated with both personnel and payroll functions. Describe four data items that could be used by both functions. Describe two data items for each function that would not necessarily be needed by the other (e.g., spouse name for personnel but not payroll).

Data items used by both functions:

Personnel action forms Time sheets Payroll deduction authorizations Tax withholding forms

8-2. Why are accounting transactions associated with payroll processing so repetitive in nature? Why do some companies choose to have payroll processed by external service bureaus rather than do it themselves?

Page 32: DQ_COMPUTER

It is because the payroll process is about the company’s employees. Companies who choose to have their payroll be processed by external service bureaus is because the cost is much more lower rather do it themselves.

8-3. In this chapter, we discussed many data inputs to an organization’s production process. What are the specific data items to input to a system when adding a new raw materials inventory item? What specific data items need to be input when a worker records time spent on the production line?

An example of data item to input to a system when adding new raw materials to inventory is a materials requisition form. He or she issues a materials requisition form to acquire more material from a storeroom or warehouse where the raw materials are kept.

An example of a specific data items need to be input when a worker records time spent on the production line is a job time card. This card shows the distribution of labor costs to specific jobs or production orders. Each worker completes a job time card (usually daily or weekly), detailing the hours worked on specific operations and jobs.

8-4. What nonfinancial information would be important for an AIS to capture about a manufacturing firm’s production process?

Inadequacies in financial performance measures have encouraged companies to consider nonfinancial measures when evaluating performance. Example of nonfinancial information is sustainability and service performance.

8-5. What are the basic concepts of lean manufacturing? What concepts are the root of lean production and lean manufacturing?

Lean manufacturing involves making the commitment to eliminate waste throughout the organization (not just in production), which is a philosophy that is often attributed to the Toyota Production System (TPS). The TPS essentially focuses on elimination or reduction of non-value-added waste to improve overall customer value and to increase the profitability of the products or services that the organization offers. So we might say that lean manufacturing developed through the concepts of JIT as well as Total Quality Management.

8-6. Find an example of a firm that is using lean manufacturing concepts. Has the company realized any improvements? What are they?

Sunset Manufacturing Company, Inc. is a state-of-the-art precision machining job shop serving the High Tech, Automotive, medical and aerospace markets. Over a number of years, Sunset's president, Jim Warren, developed a custom computerized shop floor data collection system to track and gather data from their traditional job costing and work flow processes.  Constantly seeking improvements in manufacturing methodology, Mr. Warren and the Sunset

Page 33: DQ_COMPUTER

management team began implementing Lean Manufacturing practices in 2002. An intense kaizen event was staged for the purpose of value stream mapping the in-house work order data collection process. Representatives from business partner companies, outside consultants as well as Sunset management and shop floor personnel participated in the rigorous review.  The outcome was a much clearer understanding of the value stream flows within the company and the decision to unplug the custom shop floor data collection system. Analysis clearly showed that it was too costly, time consuming and inaccurate a tool to support critical management business decision-making. Warren's experience at the helm of Sunset had taught him that misleading data can result in disastrous management decision making. Reflecting on the insights revealed by several kaizen events, the Sunset team decided to develop, test and implement a "Kanban" work order card system that has proven to be a dramatic success and Sunset was well on its way to becoming a fully Lean company.  

8-7. Can you find an example of a firm that is using lean production concepts that are supported by lean accounting? How are they doing?

Sunset Manufacturing Company, Inc. is a state-of-the-art precision machining job shop serving the High Tech, Automotive, medical and aerospace markets. Over a number of years, Sunset's president, Jim Warren, developed a custom computerized shop floor data collection system to track and gather data from their traditional job costing and work flow processes.  Constantly seeking improvements in manufacturing methodology, Mr. Warren and the Sunset management team began implementing Lean Manufacturing practices in 2002. An intense kaizen event was staged for the purpose of value stream mapping the in-house work order data collection process. Representatives from business partner companies, outside consultants as well as Sunset management and shop floor personnel participated in the rigorous review.  The outcome was a much clearer understanding of the value stream flows within the company and the decision to unplug the custom shop floor data collection system. Analysis clearly showed that it was too costly, time consuming and inaccurate a tool to support critical management business decision-making. Warren's experience at the helm of Sunset had taught him that misleading data can result in disastrous management decision making. Reflecting on the insights revealed by several kaizen events, the Sunset team decided to develop, test and implement a "Kanban" work order card system that has proven to be a dramatic success and Sunset was well on its way to becoming a fully Lean company.  In 2004, Mr. Warren and Sunset participated in the startup of the Lean Accounting SIG (special interest group) in conjunction with the local lean consortium, Northwest High Performance Enterprise Consortium (NWHPEC). The purpose of the Lean Accounting SIG was to facilitate

Page 34: DQ_COMPUTER

learning about lean accounting principles, methodologies and implementation strategies among business associates advocating lean manufacturing principles. Because of what they did, the company is doing well financially speaking.

8-8. Are the inputs and outputs of a production process likely to be different for a home builder than for a cement company? How?

No. There will be differences in some aspects but it will be most likely similar to each other. It is because both industries will use the same inputs and outputs.

8-9. There are many vertical market industries with special accounting information needs apart from the industries discussed in this chapter. Identify three additional vertical market industries. What are the unique characteristics of these industries that affect their AISs?

Vertical Market Industries: (a) Banking – this industry is under strict supervision of the government and interest rates (b) Real Estate – value of land (c) Jewelry – high value of inventory

8-10. Discuss specific steps you would take as a manager to ensure that a business process reengineering effort is successful.

I would first start out by evaluating each person that performs the jobs and see if it were possible to allow one person to perform all of the tasks as the others performed another task. Design a specific department for specific activity and have a couple of employees only handle these. Then train the employees. If possible, seek out advices from experts.

Chapter 9

9-1. What are the primary provisions of the 1992 COSO Report? The 2004 COSO Report?

The 1992 COSO Report is important because it established a common definition of internal control for assessing control systems, as well as determined how to improve controls. According to the report, controls can serve many important purposes, and for this reason, many businesses look at internal control systems as a solution to a variety of potential problems (such as dealing with rapidly changing economic and competitive environments, as well as shifting customer demands and priorities). According to the COSO report, an internal control system should consist of these five components: (1) the control environment, (2) risk assessment, (3) control activities, (4) information and communication, and (5) monitoring.

Meanwhile, the 2004 COSO Enterprise Risk Management—Integrated Framework, focuses on enterprise risk management (ERM) and builds upon the 1992 COSO Internal Control—Integrated Framework (ICIF). The ERM Framework includes the five components of ICIF (control environment, risk assessment, control activities, information and communication, and

Page 35: DQ_COMPUTER

monitoring) and adds three additional components: objective setting, event identification, and risk response

9-2. What are the primary provisions of COBIT?

A framework for IT management; Provides managers, auditors, and IT users a set of generally accepted measures, indicators, processes, best practices to maximize benefits of IT and develop appropriate IT governance and control

9-3. Why are the COSO and COBIT frameworks so important?

COSO and COBIT are - among other things - control frameworks. COSO focuses on controls for financial processes, and COBIT focuses on IT.

9-4. Briefly discuss the interrelated components that should exist within an internal control system. In your opinion, which component is the most important and why?

The control environment, risk assessment, control activities, information and communication, and monitoring are the five interrelated components that make up an internal control system. The control environment establishes the tone of a company, which influences the control awareness of the company’s employees. Risk assessment is about auditor’s evaluation of the desirability of IT-related controls for a particular aspect of business risk. Control activities are the policies and procedures that management develops to help protect the firm’s assets. Information and communication is about comprehensive evaluation of an organization’s ability to send, receive and share information with various audiences within the organization. Monitoring is an on-going process usually directed by management to ensure processes are working as intended. For me, risk-assessment is important among all because it dictates on what the entity has and need to do.

9-5. Why are accountants so concerned about their organization having an efficient and effective internal control system?

It is for audit purposes. Usually, accountants/auditors need to know if an entity’s internal is effective or efficient in order to know what things are needed to do in auditing the entity.

9-6. Discuss what you consider to be the major differences between preventive, detective, and corrective control procedures. Give two examples of each type of control.

Preventive control activities aim to deter the instance of errors or fraud. Example: thorough documentation and authorization

Detective control activities identify undesirable "occurrences" after the fact. The most obvious detective control activity is reconciliation.

Corrective controls are control procedures within a company’s internal control designed to remedy problems discovered through detective controls.

Page 36: DQ_COMPUTER

9-7. Why are competent employees important to an organization’s internal control system?

It is simply because an employee can directly affect an organization’s internal control system.

9-8. How can separation of duties reduce the risk of undetected errors and irregularities?

Imagine having one person do two important work like being custodian and the one depositing daily sales on bank. If one person will do such tasks, it may impair their credibility and it is more likely that the employee will be tempted to do fraudulent activities.

9-9. Discuss some of the advantages to an organization from using a voucher system and pre-numbered checks for its cash disbursement transactions.

Under a voucher system, the employee prepares a disbursement voucher that identifies the specific vendor, lists the outstanding invoices, specifies the general ledger accounts to be debited, and shows the net amount to pay the vendor after deducting any returns and allowances as well as any purchase discount. Like the receiving report, we depict paper copies, but both of these processes are most likely electronic in today’s organizations.

9-10. What role does cost-benefit analysis play in an organization’s internal control system?

It helps the management to quantify the value of the cost they incur and the benefits they derived in incurring cost for their internal control system.

9-11. Why is it important for managers to evaluate internal controls?

From a cost-benefit viewpoint, therefore, managers are sometimes forced to design and implement control procedures for specific areas of their company that are less than ideal. These managers must learn to live with the fact that, for example, some irregularities may occur in their organizational system, and these problems will not be detected by the internal control system. Another approach to cost-benefit analysis attempts to quantify the risk factor associated with a specific area of a company.

9-12. Why did COSO think it was so important to issue the 2009 Report on monitoring?

It is because it identifies three broad principles for monitoring: establish baseline, design and execute procedures, assess and report results

Page 37: DQ_COMPUTER

Chapter 10

10-1. What is a security policy? What do we mean when we say organizations should have an integrated security plan?

Security policy is a comprehensive plan that helps protect an enterprise from both internal and external threats. The policy is based on assessment of the risks faced by the firm. An integrated security plan means combining a number of logical and physical security technologies, including firewalls, intrusion detection systems, content filtering, vulnerability management, virus protection, and virtual private networks.

10-2. What do we mean when we talk about convergence of physical and logical security? Why might this be important to an organization?

A sound security posture must be designed holistically. Having a single governance team, organization, or cross departmental virtual team accountable for the overall design of the policies and procedures helps ensure that the needs of all stakeholders are met and that the deployment allows for the flexibility to manage across silos without negatively impacting the ability to create the desired security posture. The integration of physical and logical security domains allows you to better secure your assets.

Page 38: DQ_COMPUTER

Once an organization has implemented a converged security solution, there are several different ways it can be used, each with its own benefits. Some ways that converged solutions have been used in the past include:

Network Access: Organizations can set network access policies for the organization, with conditions where a user is granted both network and remote access only with a valid ID badge; a user is granted network access only if he has logged in within a specified time after entering the facility; or a user is granted network access only upon entry through a specific door or zone.

Event Management: With converged security solutions where companies can see where an employee is or isn't, administrators can respond more promptly to security events such as when a network account is being accessed when the employee is not present in the facility; or alternatively, when a remote account is being accessed while the user in question is in the building; and in the case that a terminated employee attempts to gain network or remote access.

Access Reports: Organizations will be able to track each employee's network and remote access history and compare them against facility entry records. This has proven useful for providing a complete timeline, establishing a history of how and when a user entered a building, logged onto a network and what applications were accessed. This comprehensive audit trail is extremely useful for investigating breaches or leaks.

10-3. What guidance or framework would you use to establish IT governance if you were a senior executive in a firm? If you were a midlevel IT manager who was designing IT general controls? A manager who was responsible for identifying the appropriate application controls?

10-4. What controls must be used to protect data that is transmitted across wireless networks?

Probably the most important control for wireless technology is installing a virtual private network (VPN), which is a security appliance that runs behind a company’s firewall and allows remote users to access entity resources by using wireless, handheld devices. The risk of unauthorized access to data through electronic eavesdropping is minimized by using data encryption. It can be used to prevent a company’s competitors from electronically monitoring confidential data transmissions. Encryption converts data into a scrambled format prior to their transmission and converts the data back into a meaningful form once transmission is complete.

10-5. Why is business continuity planning so important? Identify several reasons why testing the plan is a good idea.

Critical services or products are those that must be delivered to ensure survival, avoid causing injury, and meet legal or other obligations of an organization. Business Continuity Planning is a proactive planning process that ensures critical services or products are delivered during a disruption.

A Business Continuity Plan includes:

Plans, measures and arrangements to ensure the continuous delivery of critical services and products, which permits the organization to recover its facility, data and assets.

Page 39: DQ_COMPUTER

Identification of necessary resources to support business continuity, including personnel, information, equipment, financial allocations, legal counsel, infrastructure protection and accommodations.

Having a BCP enhances an organization's image with employees, shareholders and customers by demonstrating a proactive attitude. Additional benefits include improvement in overall organizational efficiency and identifying the relationship of assets and human and financial resources to critical services and deliverables.

Testing verifies the effectiveness of your plan, trains plan participants on what to do in a real scenario and identifies areas where the plan needs to be strengthened.

10-6. What is backup and why is it important when operating an accounting system?

Backup is the activity of copying files or databases so that they will be preserved in case of equipment failure or other catastrophe. Backup is usually a routine part of the operation of large businesses with mainframes as well as the administrators of smaller business computers.

Accounting systems contain confidential information that should be kept safe and secure at all times. The consequences of unauthorized access can be devastating--from identity theft problems to loss of irreplaceable data. When accounting data is changed or deleted on purpose or by chance, it creates havoc in the accounting department, calling into question the reliability or accuracy of all data. A standard security procedure with accounting systems is to back up data and save the backup in a safe place outside the premises. The point is that if something happens to the system, such as fires, floods or other losses, data is safe and can be restored. A good security measure is to perform overnight backups as well as to restore backups once in a while to make sure the data is safe and usable.

10-7. Discuss some of the unique control risks associated with the use of PCs and laptop computers compared to using mainframes. List what you consider to be three of the most important control procedures that should be implemented for PCs. For each control procedure, give your reason for including this procedure as an important control.

PCs are relatively inexpensive. Therefore, it may not be cost-effective for a companyto go to elaborate lengths to protect them. What companies should do is use inexpensive,yet effective, control procedures for PCs. It should be noted that because of the compactnature of laptop PCs, theft of these assets has become a big problem for both corporateentities and government agencies.

Control Procedures How to use the procedure

Identify your laptop

•Which model? What configuration? What is the serial number? Are there any other unique identifiers? Without these details, law enforcement agencies, airlines, hotels, and so on, have little chance of retrieving your company’s stolen laptop.

Page 40: DQ_COMPUTER

Use non breakable cables to attach laptops to stationary furniture

• For example, in a hotel room be sure to use a cable or other security device to attach the laptop to some stationary object such as a desk or some other piece of heavy furniture.

Load Antivirus software onto the hard disk

• Automatically perform an Antivirus scan whenever the laptop is turned on or whenever a diskette is inserted.• Have Antivirus scanning software check all changed or new files.• Keep Antivirus software current. Keep virus definitions current

Back up laptop information

• Never keep backups in the laptop case.• While traveling, keep backup diskettes in a pocket or briefcase.• If possible, back up to your company’s internal network via modem, when you are out of the office.• Back up frequently and test regularly to ensure data integrity.

Among the listed control procedures above, I consider to be three of the most important control procedures that should be implemented for PCs are as follows:

1. Identify your laptop/PCThis helps law enforcement agencies, airlines, hotels, and so on, to have a better chance of retrieving your company’s stolen laptop/PC.

2. Load Antivirus software onto the hard diskIt prevents the corruption of the files in your drive and helps keep the system operates efficiently and functionally.

3. Back up laptop informationThis enables you to maintain a record of your files if your laptop/PC get lost or stolen.

10-8. Jean & Joan Cosmetics has a complete line of beauty products for women and maintains a computerized inventory system. An eight-digit product number identifies inventory items, of which the first four digits classify the beauty product by major category (hair, face, skin, eyes, etc.) and the last four digits identify the product itself. Identify as many controls as you can that the company might use to ensure accuracy in this eight-digit number when updating its inventory-balance file.

10-9. Explain how each of the following can be used to control the input, processing, or outputof accounting data: (a) edit tests, (b) check digits, (c) passwords, (d) activity listings, and (e) control totals.

a. Edit testsEdit tests examine selected fields of input data and reject those transactions (or other types of data input) whose data fields do not meet the preestablished standards of data quality. Realtime processing systems perform edit tests during the data-entry process.

Page 41: DQ_COMPUTER

Edit tests can also be coordinated in what is called a redundant data check to ensure data accuracy. The idea is to encode repetitious data on a file or transaction record, thereby enabling a later processing test to compare the two data items for compatibility.

b. Check DigitsA check digit is a form of redundancy check used for error detection on identification numbers (e.g. bank account numbers) which have been input manually. It is analogous to a binary parity bit used to check for errors in computer-generated data.

c. PasswordsA password is a word or string of characters used for user authentication to prove identity or access approval to gain access to a resource (example: an access code is a type of password), which should be kept secret from those not allowed access.

d. Activity ListingsThese listings provide complete, detailed information about all changes to master files and thus contribute to a good audit trail. Organizational employees use such activity listings to trace file changes back to the events or documents that triggered these changes and thereby verify current file information or printed information as valid, accurate, and complete output.

e. Control TotalsWhen computer processing commences, the special information on the lead control record (i.e., the batch control document) is accessed first and the batch control total is stored in computer memory. A control total that involves a dollar amount is called a financial control total. Other examples of financial control totals include the sum of cash receipts in an accounts receivable application, the sum of cash disbursements in an accounts payable application, and the sum of net pay in a payroll application.

10-10. What is the difference between logical access to the computer and physical access to the computer? Why is the security of both important?

Physical security refers to any measures that an organization uses to protect its facilities, resources, or its proprietary data that are stored on physical media.Whereas, Logical security uses technology to limit access to only authorized individuals to the organization’s systems and information, such as password controls.

Both physical and logical security groups are required to maintain various roles and rule-based access. Areas such as distributed security administration, policy administration, and access control are included in both types of security. However, most companies currently have two completely separate data stores controlling access. Combining these in a common identity management system not only increases security by minimizing the impact on change management, but also saves costs in terms of compute cycles, storage, and manpower when a change request occurs.

Page 42: DQ_COMPUTER

10-11. Discuss the following statement: ‘‘The separation of duties control is very difficult in computerized accounting information systems because computers often integrate functions when performing data processing tasks. Therefore, such a control is not advisable for those organizations using computers to perform their accounting functions.’’

10-12. Discuss the role of the hash total in accounting information systems.

Control totals do not have to make sense to be useful. For example, when cash receipts from customers are processed, the manual sum of the customers’ account numbers in a batch of transactions might be computed to form a hash total. This sum is meaningless, but is useful as a check against an ‘‘internal’’ tally of this same hash total by the computer at the time of data access.

Chapter 11

11-1. The cases of computer crime that we know about have been described as just ‘‘the tip of the iceberg.’’ Do you consider this description accurate? Why or why not?

Yes, it means that we only knew a little about the computer crimes committed. What we see in an iceberg is only a portion of the iceberg, below the surface is the huge portion. In the same manner, we are not aware of how majority of these crimes are being carried-out and by whom, and keeps growing without us knowing much about it.

11-2. Most computer crimes are not reported. Give as many reasons as you can why much of this crime is purposely downplayed. Do you consider these reasons valid?

One reason for this is the fact that a large proportion of computer crime and abuse takes place in private companies, where it is handled as an internal matter. We have no laws that require organizations to report computer offenses. But the most important reason we know so little about computer offenses is because we believe that most of it is not discovered.

11-3. Why have most computer experts suggested that computer crime is growing despite the fact that so little is known about it?

Page 43: DQ_COMPUTER

One reason is the exponential growth in the use of computer resources—for example, microcomputers, computer networks, and the Internet. As more people become knowledgeable about how to use computers, more people are in a position to compromise computer systems.

Another reason why we believe computer crime is growing is because of continuing lax security. There are millions of computer users in the world, but many of them are not aware of, or conscientious about, computer security. Then, too, some users are dishonest and have a new tool with which to commit frauds. Lastly, many Web sites now give step-by step instructions on how to perpetrate computer crimes.

11-4. Does a company have the right to collect, store, and disseminate information about your purchasing activities without your permission?

A company has no right to disseminate the information about the purchasing activities of its clients without their permission. Doing so, might pave the way to trick the client and the client might incur damages because of that. Unless, required by law, the company must maintain the record private. 11-5. What enabled the employees at TRW to get away with their crime? What controls mighthave prevented the crime from occurring?

Clients of TRW included banks, retail stores, and such credit-conscious concerns as Diner’s Club, American Express, MasterCard, Visa, Sears, Roebuck and Co., and several leasing establishments. TRW advised its clients of bad credit risks on the basis of the information maintained in its databases. However, this information could be changed. The fraud began when six company employees, including a key TRW clerk in the consumer relations department, realized this fact and began selling good credit to individuals with bad credit ratings. The names and addresses of the bad credit risks were already on file. It merely remained to contact these individuals and inform them of a newfound method of altering their records. Accordingly, the perpetrators approached individuals with bad credit ratings and offered them a clean bill of health in return for a ‘‘management fee.’’

The TRW case involves two key issues: (1) the propriety of the input information used in updating a specific AIS, and (2) the protection afforded both consumer and user in the accuracy and use of credit information that is gathered by a private company. With regard to the first point, it is clear that the fraud was successful only because the perpetrators were able to enter false information into the computer system. This observation points to the importance of control procedures (e.g., the authorization and validation of credit changes) that safeguard the accuracy and completeness of file information. The second point, which involves protection of the consumer and the user of credit information, encompasses a much larger issue. In 1970, Congress passed the Fair Credit Reporting Act, which requires that an individual be told why he or she is denied credit. The consumer also has the right to contest the information maintained by the credit-rating company, although there is clearly a vast difference between the right to challenge and the right to change credit information.

Page 44: DQ_COMPUTER

11-6. What is hacking? What can be done to prevent hacking?

Computer hacking is broadly defined as intentionally accesses a computer without authorization or exceeds authorized access.

The most useful protection against hacking is encryption. Encryption also protects stored data, which are then rendered useless to a hacker even if he or she manages to gain access to files that are protected by other means. Finally, encryption is useful in networking situations because a properly encrypted password further ensures that an authentic user sent it.

Another way to identify an organization’s vulnerability to hacking is by hiring ethical hackers. These are network and computer experts who purposefully attack a secured system to help its owners find any vulnerability that could be exploited by a malicious hacker. Ethical hackers use the same methods as a malicious hacker to test a security system, but instead of taking advantage of these vulnerabilities, they report them to management.

Another helpful practice is user education—that is, making potential hackers aware of the ethics of computer usage and the inconvenience, lost time, and costs incurred by victim organizations.

11-7. What is a computer virus?

A computer virus is an attachment to other files or programs that destroys computer files, disrupts operating system activities, or damages program software.

11-8. How can educating employees help stop computer crime?

Informing employees of the significance of computer crime and abuse, the amount it costs, and the work disruption it creates helps employees understand why computer offenses are a serious matter. Studies suggest that informal discussions, periodic departmental memos, and formal guidelines are among the most popular educational tools for informing employees about computer crime and abuse. Requiring new hires to sign security statements indicating that they have received, read, and understand policy statements can also help.

11-9. What computer crimes are committed on the Internet? What assets are involved? What can be done to safeguard these assets?

Examples include thieves supplying fake credit card numbers to buy everything from investment securities to Internet access time itself, copying Web pages without permission, denying legitimate users Internet access, and posing as someone else for any number of illegal or dishonest purposes.

Because of the widespread use of computers in business, coupled with the fact that many employees travel and use laptops, employers should develop and distribute a company policy with respect to privacy. The Fair Employment Practices Guidelines suggest that these policies cover issues such as (1) who owns the computer and the data stored on the computer, (2) what purposes the computer may be used (e.g., primarily for business purposes), and (3) what uses are

Page 45: DQ_COMPUTER

unauthorized or prohibited. Further, employers should specifically identify the types of acceptable and unacceptable uses, with some examples. Another idea is to have a screen pop-up each time an employee signs on that reminds the employee of the company policy. In general, companies would benefit from legal counselon this topic.

11-10. How would you define “ethics”? What types of ethical issues are involved in computerized accounting information systems? How can organizations encourage their employees to act ethically?

Ethics, sometimes known as moral philosophy, is a branch of philosophy that involves systematizing, defending and recommending concepts of right and wrong conduct. The following are the ethical issues faced by the organization:

1. Honesty2. Protecting computer systems3. Protecting confidential information4. Social responsibility5. Acceptable use6. Rights of privacy

Organizations should (1) inform employees that ethics are important, (2) formally expose employees to relevant cases that teach them how to act responsibly in specific situations, (3) teach by example, that is, by managers acting responsibly, and (4) use job promotions and other benefits to reward those employees who act responsibly.

11-11. The Rivera Regional Bank uses a computerized data processing system to maintain both its checking accounts and its savings accounts. During the last three years, several customers have complained that their balances have been in error. Randy Allen, the information systems bank manager, has always treated these customers very courteously and has personally seen to it that the problems have been rectified quickly, sometimes by putting in extra hours after normal quitting time to make the necessary changes. This extra effort has been so helpful to the bank that this year, the bank’s top management is planning to select Mr. Allen for the Employee-of-the-Year Award. Mr. Allen has never taken a vacation. Comment.

Selecting Mr. Allen for the Employee-Of-The-Year award will help the employee in motivating himself to be better at what his doing. The company should also encourage other employees to take part in maintaining the integrity of the company. The company should also conduct an investigation as to why the complaints of the customers are taking effect.

Page 46: DQ_COMPUTER

Chapter 12

12-1. Distinguish between the roles of an internal and an external auditor. Cite at least twoexamples of auditing procedures that might reasonably be expected of an internal auditorbut not an external auditor. Which type of auditor would you rather be? Why?

Internal auditors can more easily specialize in knowledge about their particular organization’s hardware, operating system platform, and application programs. On the other hand, an external auditor is likely to audit the information systems of many different client organizations. The external information systems auditor may or may not be part of the regular financial audit team.

12-2. How does information technology auditing differ from financial auditing? Make a list of theskills you think are important for financial auditors and for IT auditors. Do you think allauditors should have all the skills on both lists? Why or why not?

Informationtechnologyauditingconcernsitselfwiththeevaluationoftheinformationtechnologyinfrastructureofafirm,itspoliciesandoperations.ThisisalsotakenintoaccountinfinancialauditbutitismoreextensivelyevaluatedinITauditratherthaninfinancialaudit

FinancialAuditor

Page 47: DQ_COMPUTER

Theabilitytointerpretandanalyzefinancialfiguresandinformation Excellentmathematicalabilities Excellentcommunicationskillsbothwrittenandverbal(communicatingeffectivelywithclients

isamajorpartofthejob) Timemanagementskillsandtheabilitytoworkwelltotightdeadlinesandunderpressure Ahighlevelofaccuracyandgoodattentiontodetail Theabilitytomanageowntasksandtoworkeffectivelyaspartofawiderteam Goodbusinessacumen Knowledgeof(oftencomplex)financialsystems Awillingnesstotreatsensitivedatawithdiscretion Goodproblemsolvingabilities Abilitytocoordinateanddelegateworktoothermembersoftheteam Abilitytowritereports Theabilitytojuggleseveraltasksatonce Goodmanagerialskills

Informationtechnologyauditor

Basicauditskills. Desiretounderstandtechnology. Educationalbackgroundincomputerscienceorrelatedfield. Communicationskills. AbilityandwillingnesstotrainothersingeneralITauditskills. Theabilitytounderstandnewtechnologiesinashort-timeperiod. Detail-Oriented Professional TechSavvy Certified

Itisnotnecessarilythatanaspiringauditormusthaveallthisskills.Itdependsonwhatfieldsofauditingyouaregoingtoundertakebecausesomeskillsaremoreusefulinonefieldbutmaynotbeasusefulinanotherfield.Butwecan’tdenythathavingalltheseskillsisaverygreatadvantageamongothers.

12-3. Describe the differences between general-use software and generalized audit software. Howmight you use spreadsheet software, database software, and word processing software inconducting an audit of fixed assets?

General-usesoftwaresaresoftwaresthatcanbeusedinmostfieldsofworkandcanbeusedbymostindividuals.Whilegeneralizedauditsoftwaresaremostlyusetoaidauditorswiththeirauditingproceduressothesetypesofsoftwarearelimitedtothefieldofauditandmostlyusebyprofessionals.

Page 48: DQ_COMPUTER

Intheauditoffixedassets,spreadsheetsoftware,databasesoftwareandwordprocessingsoftwarecanbeusedinmakingreports,obtainingdataandinformation,compilingandsummarizingdataandinformationandperformingstraightforwardcalculations.

12-4. IT auditors need people skills as well as technical skills. One such skill is the ability tointerview effectively. Discuss some techniques or tools that might help an interviewer getthe best information from an interviewee, including sensitive information.

Beingmorepersonableandabletobuildtrustquicklywillgetpeopletoopenuptoyouanddeliverinformationtheymaynothaveotherwise.Learningskillsonhowtointerrogatewouldhavereadbodylanguageandsignshiddenbetweenthelinesoflies.Learningtheaspectsofthepositionthepersonworksinwillhelptheintervieweraskbetterquestionsanddeliverywhat-ifs.

12-5. Describe how an auditor might use through-the-computer techniques such as test data, an integrated test facility, parallel simulation, or validation of computer programs to accomplish audit objectives relative to accounts payable.

Testdatawillallowanauditortochecktherangeofexceptionsituationsandcomparetheresultswithapredeterminedsetofanswersonanauditworksheet,suchasinvaliddatesanduseofalphabeticdatainnumericcodes.Anintegratedtestfacilitywillallowanauditinanoperationalsettingbyusingartificialtransactionsandcompanies,suchaspaymentstovendorsandshipments/ordersfromvendors.Parallelsimulationallowstheauditortorunlivedatainsteadoftestdatainasecondsystemthatduplicatestheclientsystemtolookfordifferences,suchaspaymentstovendorsonlysystemandnottheentireaccountspayableprogram.Validationallowsanauditortoguardagainstprogramtamperingwithprogramchangecontrols,programcomparison,reviewsofthesystemsoftware,validatingusersandaccessprivileges,andcontinuousauditingforreal-timeassurance.

12-6. Jose Rodriguez was the only internal auditor of a medium-sized communications firm. Thecompany used a computer for most of its accounting applications, and recently, several newsoftware packages had been implemented to handle the increased volume of the company’sbusiness. To evaluate the packages’ control capabilities, Jose performed a cost-benefit analysis and found that many of the control procedures were potentially useful but notclearly cost-effective. The problem, therefore, was what to say in his report to management.After pondering this question for some time, he decided to recommend almost all thecontrolsbasedon theideathatacompanywas “bettertobesafethansorry.” Comment on the wisdom of this idea.

Acompanyalwayswantstobesafe,butwhencostsareanissue,priorityguidanceisamust.Theauditorandthecompanyshouldinvestinacomputerizedauditingsoftwaretohelpaudit.Thecontrols,eventhoughallbeneficial,shouldstillbeportrayedinahierarchytoshowwhichonesaretechnicallyworthmore(riskassessment).Theauditorshouldevaluatethosecontrolprocedures(systemsreview)andthenevaluatetheweaknesses.ControlweaknessesinoneareaofanAISmaybeacceptableifcontrolstrengthsinotherareasoftheAIScompensateforthem.

Page 49: DQ_COMPUTER

12-7. This chapter described several third party assurance seals, including CPA WebTrust, BBB-Online, and TRUSTe. Explain the differences among them. Identify at least one other third-party assurance seal available for companies that allows them to demonstrate to theircustomers that they may be trusted in business transactions.

WebTrust is an assurance service jointly developed by the American Institute of Certified Public Accountants (AICPA) and the Canadian Institute of Chartered Accountants (CICA). WebTrust relies on a series of principles and criteria designed to promote confidence and trust between consumers and companies conducting business on the Internet. Public accounting firms and practitioners, who obtain a WebTrust business license from the AICPA or CICA, can provide assurance services to evaluate and test whether a particular web site meets any one of the Trust Services principles and criteria. The WebTrust seal of assurance is placed on the organization's web site following the engagement and signifies the practitioner's unqualified opinion.

Independent third parties may provide business users and individual consumers with some level of comfort over their Internet transactions. The comfort level varies with the type of assurance services offered. In some cases, third-party assurance is limited to data privacy. The TRUSTeassurance seal is an example of privacy assurance. TRUSTe is a nonprofit organization that issues a privacy seal. TRUSTenow offers a specialized seal to businesses that assures customers that a company will not spam them or misuse their e-mail address.

Other assurance services offer different kinds of protection. Consumers and business partners are not only concerned about privacy and security of data transmissions. They also worry about the business policies of an Internet company, its ability to deliver goods and services in a timely fashion, its billing procedures, and its integrity in using a customer’s e-mail address. The Better Business Bureau’s BBB Online seeks to verify the business policies of Internet businesses.

Chapter 13

1. Discuss the major differences between the planning, analysis, and design phase of a systems study.

Planning phase involves organizing a systems study team, performing a preliminary investigation of the existing system, and developing strategicplans for the remainder of the study while Analysis phase involves analyzing the company’s current system to identify the information needs, strengths, and weaknesses of the existing system and the Design stage is where the organization eliminate (or minimize) the current system’s weak points while preserving its strengths.

Page 50: DQ_COMPUTER

2. What is a steering committee? Discuss its role in a systems study performed by a consulting firm.

Steering Committee is the one appointed by the organization’s top management to work with the study team to perform its tasks. Ideally, the committee includes top management personnel—for example, the controller, the vice president of finance, the top-level IT manager and the chief executive officer of the company. The steering committee come up with a decision based on the preliminary investigation perform by the study team. The steering committee may decide to:(1) disband the study team and do nothing,(2) perform additional preliminary investigations(3) proceed to the formal systems analysis stage of the systems study.

3. A systems study team should understand three levels of corporate goals: general systems goals, top management systems goals, and operating management systems goals. If you had to select one of these categories of systems goals as the most important to the effective operation of an organization’s information system, which one would you choose? Explain the reasons for your choice.

Top Management Systems goals is the most important system goal for it the management still holds the key for the success of the organization.

4. What is the purpose of a systems feasibility evaluation? Should this activity precede or follow the preparation of a systems specifications report for computer vendor evaluation? Explain.

System Feasibility Evaluation is the first step of the Design phase of a system study, where detailed investigation of the five feasibility areas: (1) technical feasibility, (2) operational feasibility, (3) schedule feasibility, (4) legal feasibility, and (5) economic feasibility is performed by the study team. This step precedes the preparation of system specification report.

5. Discuss some of the annual cash benefits and annual cash costs that a company might have when it creates an online ordering system on the World Wide Web.

6. What is prototyping? Under what circumstances should prototyping be used? Under what circumstances should it not be used?

Prototypingmeans developing a simplified model of a proposed information system. Its product is a prototype that is a scaled-down, experimental version of a nonexistent information system that a design team can develop cheaply and quickly for user-evaluation purposes.

Page 51: DQ_COMPUTER

7. What is the purpose of a systems specifications report? In what ways, if any, do the data included in this report differ from the data accumulated by the design team during their feasibility evaluation work?

After the design team completes its work of specifying the inputs, outputs, andprocessing requirements of the new system, the members will summarize their findings in a systems specification report. This report is submitted to the steering committee for review, comment and approval of the detailed system design. The system specification report is more detailed than the system feasibility evaluation work.

8. When implementing a new computer system, two required activities are (1) establishing controls and (2) convert data files. What is the rationale for performing activity 1 before activity 2?

Controls reduce the risk of undetected errors and irregularities in the input, processing, and output stages of data processing work.Organization must install control procedures that safeguard its assets, ensure the accuracy and reliability of accounting data, promote operating efficiency,and encourage employee compliance with prescribed managerial policies. Again, these controls should be built into a system rather than added later.

9. Three methods for implementing a new system in an organization are direct conversion, parallel conversion, and modular conversion. Discuss the advantages and disadvantages of using each of these three systems implementation methods.

Direct conversion immediately discontinuing use of the old system and letting the new system ‘‘sink or swim.’’ An alternative is parallel conversion, where the organization operates both the new and the old system for some period of time. Another choice is modular conversion, where the new system is implemented in stages, one process or module at a time.

10. What is a PERT chart? What is a Gantt chart? Discuss the advantages and disadvantages of using PERT network diagrams versus Gantt charts for planning and controlling the activities involved in implementing an information system.

Program Evaluation and Review Technique (PERT) and Gantt charts, help managers schedule and monitor the activities involved in large projects, such as the implementation of a large-scale information system.

PERT is a useful project management tool because of its ability to help managers identify critical paths and areas where slack time occurs while Gantt charts are useful for both scheduling and tracking the activities of systems implementation projects because actual progress can be indicated directly on the Gantt chart and contrasted with.

11. What is the purpose of follow-up in a systems study? Describe some of the specific activities that the management implementation team would perform in their follow-up work.

Page 52: DQ_COMPUTER

Companies conduct follow-up studies to determine whether the new system is successful and, of course, to identify any new problems with it. Finally, businesses must maintain the system, which means that they correct minor flaws and update the system as required.

(A) Prepare the physical site. An organization must have physical space for any new hardware and personnel.

(B) Determine functional changes. Whenever a company makes changes to a major accounting system, it must also consider the effects of such changes on its reporting structure and personnel relationships.

(C) Select and assign personnel. Because the design team has developed detailed specifications for the new system, the organization should now have a firm idea about the job descriptions of system users.

(D) Train personnel. Both the implementation team and computer vendors can help train company employees to work with the new system, while seminars can acquaint other employees with the new system’s advantages and capabilities.

(E) Acquire and install computer equipment. After preparing the physical site location for the new computer system, the company must acquire computer equipment such as microcomputers, Web servers, routers, modems, and printers from outside vendors.

(F) Establish internal controls. Organization must install control procedures that safeguard its assets, ensure the accuracy and reliability of accounting data, promote operating efficiency,and encourageemployee compliance with prescribed managerial policies.

(G)Convert data files. When converting to a new system, an organization may have to convertitsdatafilestoalternate,moreusefulformats.Thisactivityisalsocommon when merging two system.

(H) Acquire computer software. The implementation team must also install the software thatwasacquiredordevelopedfor the project.

(I) Test computer software. Programs must be tested regardless of where they came from to ensure day-to-day processing accuracy and completeness.

(J) Convert to the new system.

12. Discuss the two major ways that a company’s software can be acquired. Which of these ways for acquiring software do you recommend? Explain your reasoning.

The company may choose between customizing an AIS that suits their system or they can just buy a software package as their new system. I suggest that a company just buy a new software with the consultation of an IT expert that fits the company’s system.

13. What is the difference between business process outsourcing (BPO) and knowledge process outsourcing (KPO)? Why do firms outsource their IT function?

Knowledge process outsourcing (KPO) an approach where an organization chooses to have some of its functions and activities related to research and acquisition of knowledge performed by an

Page 53: DQ_COMPUTER

external organization. On the otherhand, companies outsource business processes such as human resources,finance and accounting, customer services, learning services and training, and information technology.

Chapter 14

1. What are intranets? What are extranets? Why are intranets and extranets important to accountants?

Intranets networks using the same software as the Internet, but which are internal (for communications purposes) to the companies that created them while Extranets enable selected outside users to access corporate intranets. Users connect through the Internet itself via passwords or private data communications channels.

2. What are blogs? How are they used? Who is using them?

Web logs or blogs are collaboration tools that allow users with Web browsers and easy-to-use software to publish a personalized diary online. Blogging introduces a new way to create, share, and leverage knowledge in an organization and therefore can be valuable to accountants. Enterprise blogs provide companies with easy-to-use tools to manage internal and external information, which in turn affects relationships with customers, partners, and investors, as well as internal decision makers.

3. What is hypertext markup language? How does it differ from XML and XBRL? (Note: for a more comprehensive description of the differences, you may want to search the Internet.)

Page 54: DQ_COMPUTER

Hypertext markup language (HTML) is the editing language that tells a Web browser how to display information from the World Wide Web.HTML and XML are both markup languages. HTML is a way of “tagging” text to cause it display in a certain way. By contrast, XML (eXtensible Markup Language) was designed for storing and transporting data. On theotherhand, XBRL can be used to store or transport data. What makes XBRL unique is that it is designed for business and financial data use. XBRL “tags” utilize standard taxonomies.

4. How does XBRL compare to the IDEA database?

XBRL’s standardized tags make searching for items in XBRL financial documents relatively easy. One repository of such financial information is the Security and Exchange Commission’s new interactive data and electronic applications (IDEA), which the agency unveiled in August of 2008 and now contains XBRL data for over 10,000 companies—a particularly important source of financial information and a particularly important reason why standardized reporting is useful.

5. Describe some important uses of e-commerce and explain why it is important toaccountants.

E-commerce involves buying or selling goods and services electronically. This activity can be between two businesses, between a profit-seeking company and a governmental entity, or between a business and a customer. In contrast, e-business goes beyond e-commerce and deep into the processes and cultures.

7. What are electronic payments? How are they different from credit card payments?

Electronic payments (e-payments) is a faster, easier, and safer way for both customers and sellers to handle online transactions. The e-payment service acts as a trusted intermediary because it collects a payment from a buyer and pays that amount to the seller.

8. What is electronic data interchange? Why do companies use EDI?

Electronic data interchange (EDI) means transmitting information over high-speed data communication channels.Some companies prefer using EDI than Internet because of its potential advantage to transmit business documents electronically, avoiding the use of internet.

9. Most retail sales Web sites require customers to use their credit cards to make purchases online. How comfortable are you in providing your credit card number in such applications? Why do you feel this way?

Giving away your credit card number is too risky. There is a possibility that your credit card number is unsecured and place in the wrong hands. Having such financial transactions with someone hiding behind the Internet causes this feeling.

10. What is click fraud? Who benefits and who loses when click fraud occurs?

Page 55: DQ_COMPUTER

Click fraud is a growing e-commerce problem in which dishonest managers inflate the number of clicks viewers make on an advertisement on a Web page and therefore bill the linked company for more referrals than actually occurred.

10. What is spamming? How is spam related to accounting information systems? Should allspamming be illegal? Why or why not?

Spams are annoying, unsolicited e-mail messages that clog your e-mail inbox. It is also distracting, often illegal, and increasingly costly to organizations. Yes, spams should be considered illegal because they are often used to mislead people and lead to theft.

11. What are Internet firewalls and proxy servers? How are they created? How do businesses use them for Internet security?

A firewall guards against unauthorized access to sensitive file information from external Internet users while a proxy server is a network server and related software that creates a transparent gateway to and from the Internet and controls Web access. The both of them are used to protect company’s files against unwarranted intrusions from external parties.

Proxy server has the ability to funnel all incoming and outgoing Internet requests through a single server. This can make Web access more efficient because the proxy server is specifically designed to handle requests for Internet information. It can also examine all incoming requests for information and test them for authenticity.

12. What is data encryption? What techniques are used for data encryption?

Data encryption is scrambling the data in a message in a systematic way in order to prevent competitors from electronically monitoring confidential data transmissions.

13. Describe and contrast the three types of authentication. Can you think of a business situation where someone would need to use a combination of all three levels to gain access to information?

The three types of authentication are based on (1) what you have,(2) what you know, and (3) who you are. What you havemay be a plastic card that provides you physical access to information or a restricted area. Examples are your ATM card, debit card, or employee card that gives you access to certain premises. What you know refers to unique information you possess, such as a password or your mother’s maiden name.Finally, you can authenticate who you arewith a unique physical characteristic such as your fingerprint or the pattern of the retina in your eye. As you might guess, using security that forces a user to prove who they are is the highest level of authentication. The use of ATM machine requires combination of techniques but necessarily all.

Page 56: DQ_COMPUTER

14. What are digital signatures? Why do businesses use them? How can businesses use a digital certificate for Internet security?

Digital signatures are used to authenticate documents (such as purchaseorders) by including a portion of a document’s message in an encryptedformat (which reflects the digital signature).The use of digital certificate is an another authenticating technique that is issued by an independent third party called a certificate authority (e.g., Thawte or VeriSign). The certificates themselves are signed documents with sender names and public key information. Customers can also use digital certificates to assure themselves that a Web site is real.

15. Analysts claim that businesses can increase sales on the Internet, but not profits. What evidence does this chapter provide to support or refute this claim? Discuss.

Business transactions held through Internet truly increases sales but this require additional expenses like maintenance of a Web site (cloud transactions) are costly enough and affects the company’s profit.

Chapter 15

1. Which accounting software features are likely to be the most important for the following businesses? Search the Internet for an example of an AIS that you would recommend for each of these owners and include your rationale for that product.

a. A boutique shop that sells trendy ladies clothingb. A small business specializing in custom golf clubs, replacement shafts for clubs,

replacement grips for clubs, and similar repairsc. A local CPA firm with three partners, five associates, and two administrative employeesd. A pet breeder that specializes in Burmese kittense. A business that sells and rents Segways in Washington, DC, that is located on

ConstitutionAvenue, near the Lincoln Memorialf. A high-end men’s clothing business that has four stores that are all located in the same

largemetropolitan city (56 employees), and the owner is contemplating additional locationsfor stores in nearby cities

2. The difference between the price tag for middle-market accounting software versus an ERP system can be millions of dollars. What can these high-end systems do that the less expensive enterprise accounting packages cannot?

High-end systems has the ability to process transactions in multiple concurrencies and has the ability to split commissions among multiple salepersons. Large-scale software allows customers o choose from array of deployment options.

3. Discuss the differences between traditional ERP and extended enterprise systems.

Traditional ERP systems focus on internal data, generated primarily by internal and an enterprise’s own decision makers. Today’s ERP systems are extended with e-

Page 57: DQ_COMPUTER

business and other front-office capabilities. Extended enterprise systems bring customers, suppliers, and other business partners, such as investors and strategic business relations, into the picture.

4. Discuss some of the basic features of an ERP. How do these features distinguish an ERP from an integrated accounting software program?

ERP systems provide integration among many of an organization’s major business processes—for example, order processing and fulfillment, manufacturing, purchasing, and human resources functions—all of which provide data to each other and to the financial system while integrated accounting software packages may include modules or transaction groupings for general ledger, accounts receivable or sales, accounts payable or purchasing, inventory, and payroll.

5. What are some of the benefits of a centralized database architecture? What are some of thedifficulties in moving from multiple databases or files to a centralized database structure?

ERP systems architecture is configured around a central database. The database stores information about each data item just once (thus avoiding data redundancy) and makes it immediately available to all functions in an organization.

6. A new company will have no business processes in place. How would the owner go aboutselecting an appropriate AIS for the new company? Should the owner consider acquiring anERP package immediately?

No, the owner must first consult and IT expert and form a study team that will examine the company’s currents system. Indicators that company needs a new AIS; (1) Late payment of vendor invoices, which means late fees and lost cash discounts; (2) Late deliveries to customer; (3) Growth in inventories, accompanied by an increase in stockouts; (4)Slowdown in inventory turnover; (5) Increased time in collecting on receivables; (6) Late periodic reports;(7) Increasing length of time to close out books at the end of a period; (8). Managers concerned about cash flows and financial picture of organization; (9) Manager complaints about lack of information needed for decision making; and(10) Owner worries about cash flows, taxes, and profitability.

7. Find an article about a company that has adopted a business application suite. Identify the company and its basic characteristics (such as location, products, and number of employees). What are some cost savings realized by the company? Were there specific efficiencies identified as a result of the ERP implementation? Were there problems implementing the system? How long did it take for the company to complete the implementation? Were there cost or time over-runs?

Page 58: DQ_COMPUTER

8. While you are likely to purchase a middle-end accounting software package from a value-added reseller (VAR), why should you be cautious about hiring one to recommend a software package for your business?

Business owners and managers at larger firms are most likely to purchase them from a value-added reseller (VAR) or a qualified installer. VARs and qualified installers make special arrangements with software vendors to sell their programs. They also provide buyers with services such as installation, customization, and training. These services are necessary because of the complexity of the middle-range accounting programs. A VAR offers a broader array of services for more software programs than a qualified installer.

9. What are some of the consequences to a company that makes a poor decision in selecting a new AIS?

In large organizations, system redesigns (or new development work) typically involve millions of dollars, making mistakes very costly. In smaller organizations, major errors can be catastrophic, leading a firm to bankruptcy.

11. Why do businesses typically need to engage in business process reengineering when they adopt an ERP? Identify at least four key aspects of a successful BPR project.

Business process reengineering (BPR) techniques used by organizations to redesign their business processes from scratch. Key aspects of a successful BPR project: (1) Allocation of Time on Project; (2) Duration of average BPR project: 13.8 months; (3)Excellent or very good change Management directly correlates with teams that meet or exceed project objectives; (4) Successful BPR teams: oare dedicated to project , have support from top management, have a clear vision of objectives and goals; (5) Use Consultants: o as leader/key facilitator of project o to coordinate team efforts.